Initial Value Problem using Laplace Transform help?

Click For Summary
The discussion revolves around solving an initial value problem using Laplace transforms, specifically the equation y'' - 4y' + 3y = 0 with initial conditions y(0)=2 and y'(0)=8. The user attempts to apply the Laplace transform but is unsure how to proceed without using partial fractions, which their instructor has prohibited. After some back and forth, it is clarified that the inverse transform can be obtained using known relationships and properties of Laplace transforms without the need for partial fractions. Ultimately, the correct solution is identified as y = 2(e^(2t))cosh(t) + 4(e^(2t))sinh(t). The user successfully resolves their confusion and thanks the participants for their assistance.
Aristotle
Messages
169
Reaction score
1

Homework Statement


Use laplace transforms to find following initial value problem -- there is no credit for partial fractions. (i assume my teach is against using it..)

y'' - 4y' + 3y = 0 ; y(0)=2 y'(0) = 8

Homework Equations


table.JPG

Lf'' = ((s^2)*F) - s*f(0) - f'(0)
Lf' = sF - f(0)
Lf = F(s)

The Attempt at a Solution


My first attempt is of course realizing that the above equation is expressed in terms of 't'. So I must take the laplace transform on both sides.L(y'') - 4 L(y') + 3L(y) = L(0)

[((s^2)*F) - s*f(0) - f'(0)] - 4 [ sF - f(0) ] + 3 [F(s)] = 0 (substituted with the above equations)

(s^2)*F - 2s - 8 - 4s*F(s) + 8 + 3*F(s) = 0 (plugged in the initial values)

Y(s)*[s^2 - 4s + 3] - 8 = 0

Y(s) = 8 / [s^2 - 4s + 3]

Knowing that s^2 + as + b = (s + a/2 ) ^ 2 + b - ((a^2)/4)
I get: (s-2)^2 - 1 for the denominator.

Y(s) = 8 / [(s-2)^2 - 1 ]At this final step...I am not sure how I will be able to transform to Y(t) again if I am not allowed to use "Partial Fractions"? Any assistance would be truly appreciated. Thanks!
The solution for this problem is :
y = 2(e^2t)*cosht + 4(e^2t)*sinht
 
Last edited:
Physics news on Phys.org
Aristotle said:

Homework Statement


Use laplace transforms to find following initial value problem -- there is no credit for partial fractions. (i assume my teach is against using it..)

y'' - 4y' + 3y = 0 ; y(0)=2 y'(0) = 8

Homework Equations


View attachment 91182
Lf'' = ((s^2)*F) - s*f(0) - f'(0)
Lf' = sF - f(0)
Lf = F(s)

The Attempt at a Solution


My first attempt is of course realizing that the above equation is expressed in terms of 't'. So I must take the laplace transform on both sides.L(y'') - 4 L(y') + 3L(y) = L(0)

[((s^2)*F) - s*f(0) - f'(0)] - 4 [ sF - f(0) ] + 3 [F(s)] = 0 (substituted with the above equations)

(s^2)*F - 2s - 8 - 4s*F(s) + 8 + 3*F(s) = 0 (plugged in the initial values)

Y(s)*[s^2 - 4s + 3] - 8 = 0

Y(s) = 8 / [s^2 - 4s + 3]

Knowing that s^2 + as + b = (s + a/2 ) ^ 2 + b - ((a^2)/4)
I get: (s-2)^2 - 1 for the denominator.

Y(s) = 8 / [(s-2)^2 - 1 ]At this final step...I am not sure how I will be able to transform to Y(t) again if I am not allowed to use "Partial Fractions"? Any assistance would be truly appreciated. Thanks!
The solution for this problem is :
y = 2(e^2t)*cosht + 4(e^2t)*sinht

I would immediately take off marks because you do not stick with one single notation--either y or F or f, but not y as well as both F and f and in the same problem and standing for the same quantity.

Anyway, you can get the inverse transform of 1/(s^2 - 1) from your table; and you ought to know the relationship between the inverse transforms of G(s) and G(s-2). You really do not need partial fractions, and the instructor is right in forcing you to avoid them---it makes you learn to use some important properties of LTs that will be useful to you later in life.
 
Ray Vickson said:
I would immediately take off marks because you do not stick with one single notation--either y or F or f, but not y as well as both F and f and in the same problem and standing for the same quantity.

Anyway, you can get the inverse transform of 1/(s^2 - 1) from your table; and you ought to know the relationship between the inverse transforms of G(s) and G(s-2). You really do not need partial fractions, and the instructor is right in forcing you to avoid them---it makes you learn to use some important properties of LTs that will be useful to you later in life.
Using the shift theorem from Y(s) = 8 / [(s-2)^2 - 1] , I get Y(t) = 8 * L^(-1){1 / (s^2 - 1)}... 8*sinht*e^t.

I still don't know how my teacher got y = 2(e^2t)*cosht + 4(e^2t)*sinht
 
Aristotle said:
Using the shift theorem from Y(s) = 8 / [(s-2)^2 - 1] , I get Y(t) = 8 * L^(-1){1 / (s^2 - 1)}... 8*sinht*e^t.

I still don't know how my teacher got y = 2(e^2t)*cosht + 4(e^2t)*sinht

Well, for one thing, you have an incorrect ##Y(s)##: the denominator is OK but the numerator is wrong.
 
Ray Vickson said:
Well, for one thing, you have an incorrect ##Y(s)##: the denominator is OK but the numerator is wrong.

My mistake.
Never mind, I solved it. Thank you!
 
Question: A clock's minute hand has length 4 and its hour hand has length 3. What is the distance between the tips at the moment when it is increasing most rapidly?(Putnam Exam Question) Answer: Making assumption that both the hands moves at constant angular velocities, the answer is ## \sqrt{7} .## But don't you think this assumption is somewhat doubtful and wrong?

Similar threads

  • · Replies 1 ·
Replies
1
Views
1K
  • · Replies 8 ·
Replies
8
Views
2K
  • · Replies 7 ·
Replies
7
Views
2K
  • · Replies 2 ·
Replies
2
Views
2K
  • · Replies 14 ·
Replies
14
Views
3K
Replies
9
Views
2K
  • · Replies 10 ·
Replies
10
Views
2K
  • · Replies 1 ·
Replies
1
Views
1K
  • · Replies 4 ·
Replies
4
Views
3K
Replies
2
Views
2K